hesi practice exam :)

Pataasin ang iyong marka sa homework at exams ngayon gamit ang Quizwiz!

A patient had abdominal surgery and has a surgical incision, what should the nurse do next?

*Leave wound uncovered Place a dry sterile dressing over the wound *Place a sterile non-adherent dressing

A client with a pulmonary embolism is receiving streptokinase 100,000 International Units (IU)/hour for the next 24 hours. The pharmacy dispensed streptokinsease 1,500,000 in 500 mL of 0.9% normal saline (NS). The nurse should program the infusion pump to deliver how many mL/hour? (Enter numeric value only. If rounding is required, round to the nearest whole number.)

33

The nurse is documenting a client's fluid intake at lunch, which consisted of four ounces of chicken broth, one cup of water, and one half cup of flavored gelatin. How many mL of intake should be entered in the client's electronic medical record?

472 mL

After 5 months of treatment for Herpes Shingles, a patient continues to experience pain. What intervention should the nurse implement?

Assess for pain Contact HCP to prescribe pain medication Create mental health assessment

Following a disaster, which person should the nurse triage with a red tag, indicating that immediate attention is required? A. An older adult with emphysema who has a barrel chest and is coughing. B. A young adult with a history of asthma who is speaking in short sentences. C. A middle-aged man with acute coronary syndrome who has superficial facial lacerations. D. A frail elderly woman with degenerative disk disease who is complaining of back pain.

B. A young adult with a history of asthma who is speaking in short sentences. This indication suggests that the young adult is experiencing significant respiratory distress, which can rapidly lead to life-threatening hypoxia if not immediately addressed. Speaking in short sentences is a clear sign of respiratory compromise, indicating that the individual is struggling to get enough air to complete a sentence. This is a critical condition that warrants immediate intervention to secure the airway and provide respiratory support. While the other scenarios also describe individuals with potentially serious health concerns, they do not exhibit the same immediate life-threatening condition as the individual with asthma experiencing acute respiratory distress. The priority in disaster triage is to identify and initially treat those with life-threatening conditions that are amenable to immediate intervention.

During a high school class on substance abuse, a student tells the group, "If I tried cocaine, I know I could handle it. I know when to stop." What response is best for the nurse to provide? A. Addiction affects all aspects of one's life and one's family. B. An overdose of cocaine can be lethal. C. Mind altering drugs take away one's ability to make good decision D. Denial of an addiction problem is often the first response to the behavior

B. An overdose of cocaine can be lethal. This response directly addresses the student's statement by highlighting the immediate and serious risk associated with trying cocaine, even once. It underscores the potential for a life-threatening overdose, which can occur with any use of the drug, regardless of an individual's belief in their ability to "handle it" or "know when to stop." This statement provides a clear, factual consequence of cocaine use that is not dependent on the development of addiction or long-term use, making it a powerful counterargument to the student's assertion. While the other options are also true and important points in the context of substance abuse education, they may not directly counter the student's perception of being able to control drug use. Emphasizing the acute risk of lethal overdose may be more effective in challenging the student's belief in their invulnerability to the dangers of drug experimentation.

A female college student comes to the school's health clinic complaining of urinary frequency and burning with right lower back pain. Which intervention should the nurse implement first? A. Test her urine for the presence of hematuria. B. Measure her temperature and pulse rate. C. Palpate the right flank for tenderness. D. Evaluate the urine for a strong odor.

B. Measure her temperature and pulse rate This initial assessment provides vital information about the presence of infection or inflammation, which could be indicative of a urinary tract infection (UTI) or pyelonephritis, especially considering the symptoms described. Fever and an increased pulse rate can signal a systemic response to infection. Given the symptoms of urinary frequency, burning, and particularly the right lower back pain, which might suggest involvement of the kidney, assessing for fever and tachycardia is a priority to determine the severity of the condition and the need for immediate intervention.

A client with end-stage metastatic cancer has a living will stating no extraordinary measures are to be taken as death approaches, and the healthcare provider writes a "Do Not Resuscitate" (DNR) prescription. When the client begins to take gasping breaths, the nurse determines the client's oximetry meter reading is 85%. What action should the nurse implement? A. Determine if client wishes have changed B. Manually ventilate using a bag-valve-mask apparatus C. Report client's status to the healthcare provider D. Administer oxygen via nasal cannula

D. administer oxygen via nasal cannula Administering oxygen via nasal cannula is considered a basic, comfort measure rather than an extraordinary measure. It can help alleviate symptoms of dyspnea and improve the client's comfort in their final stages of life. This action aligns with the principles of palliative care, which focuses on relieving symptoms and improving quality of life for clients with serious, life-limiting illnesses. Option A, determining if the client's wishes have changed, may not be feasible or appropriate if the client is no longer able to communicate effectively due to their condition. Option B, manually ventilating using a bag-valve-mask apparatus, would be considered an extraordinary measure and contradicts the client's expressed wishes in their living will and DNR order. Option C, reporting the client's status to the healthcare provider, is always a prudent step in providing care, but it does not address the immediate need to improve the client's comfort and oxygenation as directly as administering oxygen does.

A school nurse is called to the wood shop class for a student who has a deep laceration to right forearm and is bleeding profusely. What should the nurse do while the paramedics arrive?

Have the patient lay down and elevate feet Clean the wound, apply a dressing and elevate arm

A woman gave birth to an 8 lb. newborn a few Horus ago, while in the bathroom , the nurse notes the patient has blood running down her legs. What should the nurse do?

Palpate the fundus

A client with opioid dependence arrives to the emergency department and is unresponsive with bradypnea and pinpoint pupils. What is the nurse's first intervention?

Possibilities: Establish an airway Assess neurological status

Which person should be triaged with a red tag indication as an immediate action?

Review Triage Colors

A patient with viral hepatitis and hepatic encephalopathy: What instructions should the nurse give the UAP?

Wear gloves when bathing the patient Wear a mask

The clinic medical receptionist is responsible for taking client calls relaying messages to the office nurse. Which client call should the nurse return first? A. Anginal pain relieved after taking 3 SL tablets of nitroglycerin B. Spinal osteomyelitis feeling nauseated for past 6 hours C. Hepatits A with complaint of arms and legs itching D. Casted right leg has a funny feeling tingling sensation

a. anginal pain relieved after taking 3 SL tablets of nitroglycerin The nurse should prioritize returning the call to the client who reported anginal pain relieved after taking 3 sublingual (SL) tablets of nitroglycerin. This situation could indicate unstable angina or an impending myocardial infarction, requiring immediate medical evaluation. Taking 3 nitroglycerin tablets to relieve chest pain suggests significant cardiac ischemia. It is critical to assess the client's current status and need for emergency care.

A client is recovering from an episode of urinary tract calculi. During discharge teaching, the client asks about dietary restrictions. In discussion fluid intake, the nurse should include which type of fluid limitation? A. Tea and hot chocolate B. Low-sodium soups C. Citrus fruit juices D. Over-all fluid intake

d. over-all fluid intake Instead, increasing overall fluid intake is generally advised to prevent the formation of new stones by diluting the urine, which decreases the concentration of minerals that can form stones. Among the options provided: A. "Tea and hot chocolate" contain compounds that can contribute to certain types of kidney stones. For example, tea is high in oxalates, which can contribute to the formation of calcium oxalate stones. B. "Low-sodium soups" can be part of a healthy diet, and reducing sodium intake is often recommended to prevent stone formation because high sodium levels can increase calcium in the urine. C. "Citrus fruit juices," especially those high in citrate like lemon and lime juice, can actually help prevent stone formation by binding to calcium in the urine, reducing the risk of calcium stones. Therefore, the nurse should emphasize the importance of increasing overall fluid intake to prevent future urinary tract calculi while possibly advising moderation or avoidance of specific types of drinks that may contribute to stone formation based on the type of stones the client is prone to.

The charge nurse working on a rehabilitation unit is making client assignments for 2 registered nurses (RN) that have been in the department over 3 years, and one new RN graduate who completed orientation this week. Which client should the charge nurse assign to the new RN graduate? The client A. with a T-12 spinal cord injury who is being transferred from the neurological unit B. with a head injury who is being discharged home with multiple referrals c. whose family is meeting with the rehabilitation team to discuss a treatment plan d. with a total knee replacement who has 3 hours of prescribed physical therapy

d. with a total knee replacement who has 3 hours of prescribed therapy This client's care involves more routine postoperative and rehabilitation nursing interventions, such as pain management, monitoring for signs of infection, and assisting with mobilization, which are within the scope and skill level of a new graduate. Physical therapy and rehabilitation are standard components of care for a total knee replacement, and this scenario likely involves the least complex care needs compared to the other options. Clients with spinal cord injuries (A), head injuries requiring complex discharge planning (B), or situations involving detailed family discussions about treatment plans (C) generally require more experienced nursing skills, knowledge of case management, and advanced communication skills. These situations may pose challenges that are more appropriately managed by more experienced RNs.

Which client requires immediate intervention by the nurse? A. A child with acute renal failure and hyperkalemia B. A child with cystic fibrosis who is constipated C. A toddler with chicken pox who is scratching D. An adolescent with a migraine and photophobia

A. A child with acute renal failure and hyperkalemia A child with acute renal failure and hyperkalemia requires immediate intervention by the nurse. Hyperkalemia, or high potassium levels in the blood, can be life-threatening, leading to serious cardiac arrhythmias and cardiac arrest. Acute renal failure exacerbates this risk because the kidneys are unable to excrete potassium effectively. Immediate interventions may include monitoring cardiac rhythm, administration of medications to lower potassium levels, and possibly dialysis if indicated.

When preparing a client for a paracentesis, which action is most important to delegate to the unlicensed assistive personnel (UAP) who is assisting the nurse? A. Assist the client to empty bladder B. Measure the client's abdominal girth C. Determine when the client last ate D. Obtain the client's current weight

A. Assist the client to empty bladder Emptying the bladder before a paracentesis is crucial to reduce the risk of bladder injury during the procedure. The UAP can assist the client to the bathroom or provide a bedpan or urinal, ensuring the bladder is emptied prior to the procedure. This task is within the scope of practice for UAP and directly impacts patient safety during the paracentesis.

The practical nurse (PN) caring for a client with acute heart failure (HF) and pitting edema tells the charge nurse that a verbal prescription was obtained from the healthcare provider for 1,000 mL of D5 normal saline to be administered over 4 hours. Which action should the charge nurse take? A. Clarify the prescription with the healthcare provider B. Determine if the client has crackles in the lungs C. Ensure that an IV pump is used to administer the infusion D. Assess the PN's ability to carry out the prescription

A. Clarify the prescription with the healthcare provider Administering 1,000 mL of D5 normal saline over 4 hours to a client with acute heart failure (HF) and pitting edema could exacerbate the client's condition by increasing fluid volume and potentially worsening heart failure symptoms. Fluid management is crucial in heart failure management, and intravenous fluids should be administered cautiously, especially in clients with evidence of fluid overload such as pitting edema. Clarifying the prescription with the healthcare provider ensures that the treatment plan is appropriate for the client's current condition and prevents potential harm.

An older male client is admitted with hypothermia with a core body temperature of 95° F (35°C) due to the lack of adequate heat in his home. Which findings should the nurse expect to obtain? (Select all that apply) A. Confusion B. Headache C. Hyper-alert state D. Shivering E. Cool skin

A. Confusion B. Headache D. Shivering E. Cool skin For an older male client admitted with hypothermia and a core body temperature of 95° F (35°C), the nurse should expect to observe the following findings: A. Confusion - Hypothermia can lead to altered mental status, including confusion, as the body's core temperature drops. B. Headache - A common symptom associated with hypothermia, possibly related to the body's stress response or decreased oxygenation. D. Shivering - An initial body response to cold exposure, aiming to generate heat through muscle activity. However, in severe hypothermia, shivering may eventually stop. E. Cool skin - A direct consequence of the body's reduced ability to maintain its temperature, leading to a drop in skin temperature. A hyper-alert state (C) is not typically associated with hypothermia. Instead, hypothermia often leads to decreased consciousness and a reduced physiological response rate.

When auscultating a client's breath sounds, the nurse hears vesicular sounds in the bases of both lungs posteriorly. What action should the nurse take in response to this finding? A. Conitnue with the remainder of the client's physical assessment B. Report the client's abnormal lung sounds to the healthcare provider C. Measure the client's oxygen saturation with a pulse oximeter D. Ask the client to cough and then auscultate at the site again

A. Continue with the remainder of the client's physical assessment Vesicular breath sounds are normal breath sounds heard over the majority of the lung fields. They have a soft, breezy, and low-pitched quality and are heard during inspiration and the first part of expiration. Hearing vesicular sounds in the bases of both lungs posteriorly indicates normal lung function at those sites. Therefore, the nurse should continue with the remainder of the client's physical assessment, as this finding does not indicate an abnormality that requires further immediate investigation or reporting to the healthcare provider.

After seven-day treatment with an IV antibiotic, the healthcare provider discharges a client from the hospital and writes a prescription for an oral antibiotic. In providing discharge instructions, the nurse notes that the dosage for the oral antibiotic is significantly higher than the IV antibiotic. What resource should the nurse use first in resolving the situation? A. Healthcare provider B. Medication reference guide C. Hospital pharmacist D. Nursing unit charge nures

A. Healthcare provider The healthcare provider who wrote the prescription has the clinical rationale for the dosage decision and can clarify whether the prescribed dosage is correct based on the pharmacokinetics of the oral versus IV medication, the client's condition, and any other factors that might justify the dosage difference. Direct communication with the healthcare provider ensures that any potential errors are corrected promptly and that the nurse fully understands the discharge plan to provide accurate and safe instructions to the client.

An older client who is agitated, dyspneic, orthopedic, and using accessory muscles to breathe is admitted for further treatment. Initial assessment includes a heart rate 128 beats/minute and irregular, respirations 38 breaths/minute, blood pressure 168/100 mmHg, wheezes and crackles in all lung fields, An hour after the administration of furosemide 60 mg IV, which assessments should the nurse obtain to determine the client's response to treatment? (Select all that apply) A. Oxygen saturation B. Lung Sounds C. Urinary output D. Skin elasticity E. Pain Scale

A. Oxygen saturation B. Lung Sounds C. urinary output since the client was dyspneic and using accessory muscles to breathe, improving oxygen saturation would indicate better respiratory function the presence of wheezes and crackles suggests fluid in the lung fields; improvement or resolution of these sounds can indicate the effectiveness of furosemide in reducing pulmonary congestion furosemide increases the excretion of water and salt through the urine; an increase in urinary output is a direct indicator of the drug's effectiveness in reducing fluid overload

In assessing a 10-year-old newly diagnosed with osteomyelitis, which information is most for the nurse to obtain? A. Recent recurrence of infections B. Cultural heritage and belief C. Family history of bone disorder D. Occurrence of increased fluid intake

A. Recent recurrence of infections Osteomyelitis is an infection of the bone, often caused by bacteria that have spread from another area of the body or entered the body through a wound or during surgery. Information about recent infections can help identify potential sources of the osteomyelitis and provide insight into the child's overall health and immune function. This information is crucial for guiding treatment and preventing further complications.

When initiating oxygen per mask to a client who is short of breath, the nurse hears a loud hissing sound after inserting the flowmeter into the outlet. What should the nurse do next? A. Release and reinsert the flowmeter in the wall outlet. B. Adjust the position of themes on the client's face C. Adjust the flow rate to the prescribed liters per minute D. Attach the flowmeter to a humidification canister

A. Release and reinsert the flowmeter in the wall outlet. The loud hissing sound likely indicates that the flowmeter has not been properly connected to the wall outlet, leading to a leak of oxygen. Releasing and correctly reinserting the flowmeter can secure the connection and stop the hissing sound, ensuring that the oxygen is delivered efficiently to the client. Option B, adjusting the mask on the client's face, and Option C, adjusting the flow rate to the prescribed liters per minute, are important steps in the administration of oxygen therapy but would not address the issue of the hissing sound from an improper connection at the wall outlet. Option D, attaching the flowmeter to a humidification canister, is a consideration for patient comfort and to moisten the oxygen, especially if it is being delivered at higher flow rates for an extended period. However, this action does not address the immediate problem of the hissing sound indicating a leak at the connection point.

The nurse is assessing the perianal area of a female client who states she has chronic constipation and has bright red blood on the toilet paper after having a bowel movement. Which finding should the nurse report to the healthcare provider that is most consistent with the client's complaint? A. Shiny blue skin sacs around anal opening and a linear split B. Flabby skin sac around the anal orifice that is painless C. Anus is moist, hairless, and has pigmented sphincter folds D. Presence of dried brown stool around the perianal area

A. Shiny blue skin sacs around anal opening and a linear split abnormals of perianal area: linear split--fissureflabby skin sac -- hemerrhoidshiny blue skin sac--thrombosed hemorrhoidsmall round opening in anal area--fistulainflammation or tenderness, swelling, tuft of hair, or dimple at tip of coccyx--pilonidal cystcircular red donut of tissue--rectal prolapse

AV fistula (for dialysis) normal characteristics?

Absent sound Color?

Two days after an abscess of the chin was drained, the client returns to the clinic with fever, chills, and a maculopapular rash with pruritus. The client has taken an oral antibiotic and cleaned wound today with providone-iodine solution. Which intervention should the nurse implement first? A. Review recent mediation history and allergies B. Assess airway latency and oxygen saturation C. Obtain samples for complete blood count and cultures. D. Determine if the client has a history of diabetes

B. Assess airway latency and oxygen saturation This assessment is critical because the client's symptoms could indicate an allergic reaction or a systemic infection, both of which could compromise the airway and oxygenation. Ensuring that the client has a patent airway and adequate oxygen saturation is a priority in acute care, as it addresses the most immediate life-threatening concerns. Once the airway and oxygenation are confirmed to be stable, the nurse can proceed with further assessments and interventions, such as reviewing medication history and allergies, obtaining samples for laboratory testing, and assessing for underlying conditions like diabetes that could affect the client's recovery and response to treatment.

A female client reports feeling hopeless and is unable to stop crying. She explains that she is worried about losing her job. Since the client's husband recently lost his job, she feels her employment is essential to the family's survival. To evaluate the effectiveness of cognitive-behavioral techniques which client outcome should the nurse include in the plan of care? A. Describes how the family can resolve problems B. Changes thought patterns related to problem solving C. Relates insight into problematic relationships D. Demonstrates a healthy relationship with husband

B. Changes thought patterns related to problem solving Cognitive-behavioral therapy (CBT) focuses on identifying and modifying dysfunctional thinking patterns, beliefs, and attitudes that contribute to negative emotions and behaviors. By working to change thought patterns, especially those related to problem-solving and handling stressors such as potential job loss, the client can learn to approach her situation in a more positive and constructive manner. This outcome directly measures the impact of cognitive-behavioral techniques on the client's ability to reframe her thinking and adopt more adaptive coping strategies. Option A, while problem resolution is important, doesn't specifically address the change in thought patterns that CBT aims to achieve. Option C, gaining insight into problematic relationships, and Option D, demonstrating a healthy relationship with her husband, are valuable outcomes but do not directly measure the effectiveness of cognitive-behavioral techniques focused on altering thought patterns and problem-solving approaches.

The parents of a 4 week-old infant phone the pediatric clinic to report that their infant eats well but vomits after each feeding. To differentiate between normal regurgitation and pyloric stenosis, which information is most important for the nurse to obtain? A. Level of infant's distress after vomiting B. Degree of forcefulness of vomiting episodes C. Odor and texture associated with emesis D. Position of the infant when vomiting occurs

B. Degree of forcefulness of vomiting episodes Pyloric stenosis is a condition in which the opening from the stomach to the small intestine (the pylorus) becomes narrowed due to enlargement of the muscle surrounding it. This can cause severe projectile vomiting after feedings. The degree of forcefulness of the vomiting episodes is a key indicator that can help differentiate between normal regurgitation, which is common and usually mild in infants, and the more severe projectile vomiting associated with pyloric stenosis. Projectile vomiting due to pyloric stenosis is typically forceful and may occur soon after feeding. This symptom is a critical piece of information for healthcare providers to determine the need for further evaluation and possible intervention.

A client who was involved in a motor vehicle collision is admitted with a fractured left femur which is immobilized using a fracture traction splint in preparation for an open reduction internal fixation (ORIF). The nurse determines that the client's distal pulses are diminished in the left foot. Which interventions should the nurse implement? (Select all that apply) A. Administer oral antispasmodics and narcotic analgesics B. Monitor left leg for pain, pallor, paresthesia, paralysis, pressure C. Evaluate the application of the splint to the left leg D. Offer ice chips and oral clear liquids E. Verify pedal pulses using a doppler pulse device

B. Monitor left leg for pain, pallor, paresthesia, paralysis, pressure C. Evaluate the application of the splint to the left leg E. Verify pedal pulses using a doppler pulse device these are the 5 p's of ischemia and compartment syndrome. monitoring for these signs helps in early identification of complications it's important to ensure that the splint is applied correctly and is not too tight, which could compromise blood flow and exacerbate the problem if pulses are diminished, using a doppler device can help in assessing the blood flow to the area more accurately

A client taking atorvastatin develops an increased serum creatine phosphokinase (CK) level. The nurse should assess the client for the onset of which problem? A. Excessive bruising B. Muscle tenderness C. Peripheral edema D. Nausea and vomiting

B. Muscle tenderness An increased serum creatine phosphokinase (CK) level in a client taking atorvastatin can indicate muscle damage or myopathy, which is a known side effect of statins like atorvastatin. The nurse should assess the client for muscle tenderness, weakness, or pain, which could signify the onset of statin-associated muscle symptoms (SAMS) or, in more severe cases, rhabdomyolysis.

The nurse is assessing a client who is admitted with diaphoresis and intermittent sharp abdominal pain radiating into the back and groin. While examining the abdomen, the nurse finds an abdominal mass. Which action should the nurse implement immediately? A. Assess the distal pulses B. Notify the healthcare provider C. Auscultate for bowel sounds D. Place the client in a prone position

B. Notify the healthcare provider The symptoms described, especially the presence of an abdominal mass combined with sharp, radiating pain and diaphoresis, could indicate a serious condition such as an abdominal aortic aneurysm (AAA) or another critical abdominal issue. Prompt notification of the healthcare provider is crucial for further evaluation, diagnosis, and management, potentially requiring urgent imaging studies and interventions to prevent complications like rupture or ischemia. Assessing the distal pulses (A) is important in the context of suspected AAA to check for circulation impairment, but the first step should be to alert the healthcare provider about the findings for immediate assessment and decision-making. Auscultating for bowel sounds (C) and placing the client in a prone position (D) are not appropriate immediate actions given the potential severity of the condition indicated by the symptoms and findings.

An older client who is admitted with a cerebrovascular accident is placed on a ventilator. The client's family member arrives with a durable power of attorney, and a living will that indicates the client wants no extraordinary life saving measures. Which action should the nurse take? A. Review the medical record B. Notify the healthcare provider C. Discontinue the ventilator D. Refer to the risk manager

B. Notify the healthcare provider When a client has a living will that indicates they want no extraordinary life-saving measures and they are placed on a ventilator, and there is a durable power of attorney presented by a family member, the first step is to inform the healthcare provider (physician, nurse practitioner, etc.) responsible for the client's care. The healthcare provider can then review the living will, discuss the situation with the family and the durable power of attorney holder, and determine the appropriate course of action in accordance with the client's wishes, legal guidelines, and hospital policies. Option A, reviewing the medical record, is an important step for the nurse to understand the client's condition and treatment plan, but it does not directly address the immediate need to reconcile the client's current treatment with their stated wishes in the living will. Option C, discontinuing the ventilator, is a medical decision that requires the authorization of the healthcare provider after careful review of the client's advance directives, discussion with the family/power of attorney, and consideration of the legal and ethical implications. Option D, referring to the risk manager, might be a subsequent step if there are legal or ethical concerns that need to be addressed, but the immediate action should be to communicate with the healthcare provider to ensure that the client's care aligns with their expressed wishes and legal documents.

Which action should the nurse take first when a client with systemic sclerosis (Scleroderma) exhibits symptoms of Raynaud's phenomenon? A. Initiate massage and stretching exercises B. Remove any pressure on the involved extremities C. Apply cool compresses to the involved area D. Elevate the involved extremities on pillows

B. Remove any pressure on the involved extremities Raynaud's phenomenon is characterized by episodic vasospasm of the small peripheral arteries, leading to reduced blood flow to the affected areas, often triggered by cold or stress. Symptoms include changes in skin color (white to blue to red), coldness, and numbness or pain in the affected areas. The initial nursing intervention should focus on eliminating triggers and promoting blood flow. Removing any pressure on the involved extremities is essential to avoid further restricting blood flow. Ensuring that the extremities are free from constricting clothing or jewelry helps to promote circulation. The other options, although potentially beneficial as part of the overall management strategy for Raynaud's phenomenon, are not the first actions to take. Massage and stretching exercises (A) may be helpful in improving circulation and flexibility but should be approached with caution to avoid causing trauma to sensitive areas. Applying cool compresses (C) is not advisable, as cold exposure can exacerbate Raynaud's symptoms. Elevating the involved extremities (D) may not be immediately beneficial and could potentially slow blood flow to the affected areas, depending on the individual's circulatory dynamics.

During a one-to-one session with the nurse, a female admitted for chronic depression and attempted suicide discloses experiences of sexual promiscuity and prostitution. When the nurse asks the client if she was ever sexually abused as a child, the client says, "I don't remember, but my mother ran my father off when I was five." The nurse should recognize that the client may be using which defense mechanism? A. Denial B. Repression C. Regression D. Projection

B. Repression Repression is an unconscious mechanism employed by the ego to keep disturbing or threatening thoughts from becoming conscious. In the scenario described, the client's inability to remember potential abuse could indicate that traumatic memories or experiences from her childhood are being repressed. This is particularly suggested by her vague reference to a significant family event (her mother running her father off when she was five) without directly acknowledging or recalling any abuse. Repression involves burying unpleasant feelings, memories, or experiences deep within the unconscious mind, which can influence behavior and emotional responses without the individual's conscious awareness.

A client with psoriasis returns to the clinic reporting the persistence of several silvery, scaly areas on the elbows and palms that frequently burn and sometimes bleed. Which prescription would the nurse teach the client to use for the skin condition? A. Colloidal oatmeal-based lotion B. Topical corticosteroids C. Topical analgesics D. Topical antifungal

B. Topical corticosteroids Topical corticosteroids are commonly prescribed for psoriasis to reduce inflammation, decrease the rapid turnover of skin cells, and relieve associated itching and burning sensations. They can be effective in managing psoriasis flare-ups and are often part of a comprehensive treatment plan for managing this chronic skin condition. While colloidal oatmeal-based lotion (A) can soothe the skin and provide some relief from itching, it does not address the underlying inflammation and cell turnover issues associated with psoriasis. Topical analgesics (C) may provide temporary relief from pain but do not treat the psoriasis itself. Topical antifungals (D) are used to treat fungal infections and would not be effective for psoriasis, which is an autoimmune skin disorder.

The nurse is obtaining the admission history for a client with suspected peptic ulcer disease (PUD). Which subjective data reported by the client supports this medical diagnosis? A. Frequent use of chewable and liquid antacids for indigestion B. Upper mid-abdominal pain described as gnawing and burning C. Marked loss of weight and appetite over the last 3 or 4 months D. Severe abdominal cramps and airhead after eating spicy foods

B. Upper mid-abdominal pain described as gnawing and burning Peptic ulcer disease (PUD) is characterized by ulcers forming in the stomach and upper part of the small intestine. A hallmark symptom of PUD is pain in the upper abdomen, which is often described as gnawing, burning, or aching. This type of pain typically results from the ulcer's exposure to gastric acids. It may improve after eating or worsen, depending on the ulcer's location (gastric vs. duodenal ulcer). The subjective data of upper mid-abdominal pain described as gnawing and burning directly supports the diagnosis of peptic ulcer disease.

How should the nurse instruct the parents of a 4-month-old with seborrhea dermatitis (cradle cap) to shampoo the child's hair? A. Avoid scrubbing the scalp until the scales disappear B. Use a soft brush and gently scrub the area C. Avoid washing the child's hair more than once a week D. Use soap and water and avoid shampoos

B. Use a soft brush and gently scrub the area For a child with seborrhea dermatitis (cradle cap), gentle hair care is important. Using a soft brush to gently scrub the scalp can help to loosen and remove the scales. It is also recommended to use mild baby shampoo and to wash the hair regularly. This approach helps to keep the scalp clean without irritating it further, aiding in the management and improvement of cradle cap symptoms.

While caring for a client with Amyotrophic Lateral Sclerosis (ALS), the nurse performs a neurological assessment every four hours. Which assessment finding warrants immediate intervention by the nurse? A. Asymmetrical weakness B. Weakened cough effort C. Increasing anxiety D. Inappropriate laughter

B. Weakened cough effort In ALS, respiratory muscle weakness is a significant concern as it can lead to an inability to clear secretions, increasing the risk of respiratory infections and respiratory failure. A weakened cough effort indicates that the client may not be able to effectively clear their airways, which could quickly lead to serious complications. Immediate intervention and possibly the use of cough assist devices, suctioning, or other respiratory support measures may be needed to prevent further respiratory compromise.

The mother of a 14-year-old who had below-the-knee amputation for osteosarcoma tells the nurse that her child is angry and blaming her for allowing the amputation to occur. Which response is best for the nurse to provide? A. "It is important to understand your child's needs at this difficult time" B. "I will ask the healthcare provider for a psychiatric consult for your child" C. "A reaction of anger is your child's attempt to cope with this loss" D. "This type of acting out behavior is normal for adolescent"

C. "A reaction of anger is your child's attempt to cope with this loss" This response acknowledges the emotional pain and grief the child is experiencing due to the loss of a limb and recognizes anger as a natural part of the grieving process. It validates the child's feelings without pathologizing the reaction, providing insight into the coping mechanism the child is using. This can help the mother understand that her child's anger is a normal and expected response to such a significant and life-altering event, rather than a personal attack or an abnormal behavior that requires psychiatric intervention. Option A is somewhat vague and doesn't specifically address the issue of the child's anger or provide concrete guidance or reassurance. Option B, suggesting a psychiatric consult, might be premature based on the information given and could imply that the child's reactions are not within a normal range of emotions for the situation. Option D, while attempting to normalize the behavior, is too general and doesn't capture the depth of the child's experience or provide the specific understanding that the situation is a coping mechanism for dealing with loss.

The registered nurse (RN) directs the unlicensed assistive personnel (UAP) to administer PO fluids in the ambulatory surgical center. Which adult client should the RN assign to the UAP? A. An obtunded client who requires tactile stimulation to respond. B. A sedated client asking for ice chips when questioned C. An awake client with a gag reflex who is asking for water D. A disoriented client requesting medication upon arrival from PACU

C. An awake client with a gag reflex who is asking for water This choice is the safest option because the client is awake, has a functioning gag reflex indicating the ability to swallow safely, and is requesting water, suggesting they are likely to handle oral fluids without complication. The presence of a gag reflex is a crucial indicator of the client's ability to safely swallow and not aspirate. The other options involve clients with conditions that require a higher level of nursing assessment and intervention, which are beyond the scope of practice for UAP: An obtunded client (A) requires careful assessment and monitoring for aspiration risk, which must be performed by licensed nursing staff. A sedated client (B) may have an impaired gag reflex or altered level of consciousness, increasing the risk of aspiration, thus requiring close monitoring by an RN. A disoriented client (D) requesting medication may need evaluation for their cognitive status and ability to safely take oral medications or fluids, a task requiring nursing judgment and assessment skills.

Which client should the charge nurse assign to the care of a practical nurse (PN)? A. An adolescent with a chest tube that drained 500 mL in the past 8 hours. B. An adult who is receiving chemotherapy via an implanted venous access device. C. An older client who is receiving multiple antihypertensive medications. D. An adult type 1 diabetes mellitus who is receiving a titrated insulin infusion.

C. An older client who is receiving multiple antihypertensive medications. PNs are trained to provide basic nursing care under the supervision of a registered nurse (RN) or a physician. This includes monitoring vital signs, administering oral and intramuscular medications, providing wound care, and educating patients on their health conditions. The care of an older client receiving multiple antihypertensive medications fits well within the PN's scope of practice, as it involves monitoring blood pressure, administering oral medications as prescribed, and observing for signs of hypotension or adverse effects from the medications. The other scenarios involve more complex or acute care needs that are typically outside the PN's scope of practice and require the advanced skills and judgment of an RN or specialized healthcare professional. For example, managing a chest tube with significant drainage, administering chemotherapy, and titrating insulin infusions for type 1 diabetes mellitus involve more complex care, close monitoring for complications, and adjustments based on dynamic clinical assessments, which are better suited to the expertise of an RN.

The nurse on the day shift receives report about a client with depression who was in bed most of the weekend. The nurse walks into the client's room in the morning and finds the client in bed. What intervention is best for the nurse to implement? A. Monitor the client's appetite and pattern of sleep B. Explain that staff will check on the client every 30 minutes C. Assist the client to get out of bed and involved in an activity D. Assess the client's feelings about hospital stay

C. Assist the client to get out of bed and involved in an activity Clients with depression often experience a lack of energy, motivation, and interest in activities, which can exacerbate their condition by leading to further isolation and decreased mood. Encouraging and assisting the client to get out of bed and engage in an activity can help improve their mood, provide a sense of accomplishment, and increase social interaction. Physical activity and involvement in therapeutic activities are key components of holistic care for individuals with depression, as they can positively impact mental health. While monitoring the client's appetite and pattern of sleep, explaining that staff will check on the client every 30 minutes, and assessing the client's feelings about the hospital stay are important interventions, actively engaging the client in activities offers immediate and tangible benefits to their mental health and recovery process.

A mother is concerned that her 3-year-old son wants to play with female doll figures. The child is not interested in building blocks, trucks, or other typical "boy" toys. How should the nurse respond to the mother's concern? A. Experimenting with different toys is an acceptable behavior B. Replacing female doll figures with male doll figures reinforces masculinity C. Exploring different roles in imaginary play is typical at this age D. Letting male toddlers play with female-typed toys can have negative effects

C. Exploring different roles in imaginary play is typical at this age This response reassures the mother that her child's behavior is normal and developmentally appropriate. Children at the age of 3 are in a stage of exploring the world around them, including experimenting with different roles through play. This form of play helps them understand and make sense of the world. It is a healthy part of child development and not indicative of future preferences or identity. Such exploration is crucial for cognitive and social development, allowing children to develop empathy, creativity, and a broader understanding of social roles. Option A, while also supportive, is less specific about why this behavior is acceptable and doesn't directly address the developmental aspect of role exploration through play. Option B, suggesting the replacement of female doll figures with male doll figures to reinforce masculinity, and Option D, implying negative effects from playing with female-typed toys, both promote stereotypes and do not reflect a contemporary understanding of gender development or the healthy exploration of play in early childhood.

The nurse is counseling a client who is at 6-weeks gestation and is experiencing morning sickness, but does not want to take any drugs for discomfort. Which herbal supplement if likely to help this client with nausea she is experiencing? A. Chamomile B. Ginko C. Ginger D. Peppermint

C. Ginger Ginger has been widely studied and is recognized for its effectiveness in reducing nausea and vomiting associated with morning sickness during pregnancy. It is considered safe for use in pregnancy in moderate amounts. Ginger can be consumed in various forms, such as ginger tea, ginger capsules, or even raw ginger, depending on personal preference and tolerance.

The nurse is evaluating a young child with atopic dermatitis. Which question should the nurse ask the parent while obtaining the child's history? A. What time of day does the rash appear on the body? B. Can any particular ___ be associated with onset of the rash? C. Has the child displayed any symptoms of asthma or hay fever? D. Does the child have any nausea or vomiting?

C. Has the child displayed any symptoms of asthma or hay fever? Atopic dermatitis (eczema) is often associated with other atopic conditions such as asthma and allergic rhinitis (hay fever). These conditions can share underlying immune mechanisms and genetic factors. Asking about symptoms of asthma or hay fever can help in understanding the child's overall atopic profile and may influence the management and prognosis of atopic dermatitis.

The nurse is evaluating a young child with atopic dermatitis. Which question should the nurse ask the parent while obtaining the child's history? A. What time of day does the rash appear on the body? B. Can any particular be associated with onset of the rash? C. Has the child displayed any symptoms of asthma or hay fever? D. Does the child have any nausea or vomiting?

C. Has the child displayed any symptoms of asthma or hay fever? Atopic dermatitis (eczema) is often associated with other atopic conditions, such as asthma and allergic rhinitis (hay fever). These conditions are part of the "atopic triad" and often coexist in individuals with a predisposition to allergic reactions. Asking about symptoms of asthma or hay fever can help the nurse to understand the child's overall atopic history and predisposition to allergic conditions, which is important for comprehensive management and treatment planning. This information can also provide insight into potential triggers for the dermatitis and guide preventive measures.

The nurse is obtaining a systolic blood pressure by palpation. While inflating the cuff, the radial pulse is no longer palpable at 90 mm Hg. Which action should the nurse take? A. Document the absence of the radial pulse B. Record a palpable systolic pressure of 90 C. Inflate blood pressure cuff 120 mm Hg D. Release the manometer valve immediately

C. Inflate blood pressure cuff 120 mm Hg When obtaining a systolic blood pressure by palpation, the nurse inflates the cuff until the radial pulse disappears to estimate the systolic pressure. After noting the pressure at which the radial pulse disappears (in this case, 90 mm Hg), the nurse should inflate the cuff an additional 20-30 mm Hg beyond this point to ensure that the maximum arterial pressure is exceeded, and then slowly deflate the cuff to detect the return of the pulse. The pressure at which the pulse is first palpable again during deflation is the systolic pressure by palpation. This method provides an estimate of the systolic pressure, which is particularly useful when the blood pressure is difficult to measure using auscultation.

The nurse is obtaining a systolic blood pressure by palpation. While inflating the cuff, the radial pulse is no longer palpable at 90 mm Hg. Which action should the nurse take? A. Document the absence of the radial pulse B. Record a palpable systolic pressure of 90 C. Inflate blood pressure cuff to 120 mm Hg D. Release the manometer valve immediately

C. Inflate blood pressure cuff to 120 mm Hg This technique involves inflating the cuff until the pulse disappears to estimate the systolic blood pressure. Once the pulse is no longer palpable, the nurse should inflate the cuff approximately 20-30 mm Hg above this point to ensure that the actual systolic pressure is not missed. Then, the cuff is deflated slowly, and the pressure at which the pulse first reappears is noted as the systolic blood pressure by palpation. This method provides an estimate of the systolic pressure, which is particularly useful in emergency situations or when an auscultatory blood pressure (listening with a stethoscope) cannot be obtained.

Which medication should the nurse anticipate administering to a client in the emergency department who is experiencing a hypertensive crisis? A. Mannitol B. Dopamine hydrochloride C. Labetalol hydrochloride D. Nitropress

C. Labetalol hydrochloride Both labetalol hydrochloride and sodium nitroprusside are medications commonly used to manage hypertensive emergencies. Labetalol is a beta-blocker with alpha-blocking activity, effective in quickly lowering blood pressure without causing a significant reduction in heart rate. Sodium nitroprusside (Nitropress) is a potent vasodilator that acts quickly to reduce blood pressure by relaxing blood vessels. Mannitol is an osmotic diuretic used to reduce intracranial pressure or treat acute kidney failure, not typically used for managing hypertension. Dopamine hydrochloride is a vasopressor and inotropic agent used to treat shock, low blood pressure, and heart failure, not a first-line treatment for hypertensive crisis. Given the options, labetalol hydrochloride (C) and Nitropress (D) are the medications most directly related to the management of a hypertensive crisis, with the choice between them depending on the specific clinical situation and patient needs.

A 30-year-old male client tells the nurse that about half of this diet comes from eating meat and eggs. What instruction should the nurse provide? A. Maintain protein intake but substitute fish and nuts for meat and eggs B. Increase protein intake with the additional intake of dairy products C. Maintain protein intake and increase intake of fruits and vegetables D. Decrease protein intake and eat more whole grains and vegetables

C. Maintain protein intake and increase intake of fruits and vegetables This advice encourages a balanced diet by maintaining adequate protein intake, which is important for muscle maintenance and overall health, while also increasing the intake of fruits and vegetables. Adding more fruits and vegetables to the diet can provide essential vitamins, minerals, fiber, and antioxidants, contributing to overall health and potentially reducing the risk of chronic diseases. Option A suggests substituting fish and nuts for meat and eggs, which could diversify protein sources and introduce healthy fats, but it doesn't address the need for more fruits and vegetables directly. Option B, increasing protein intake with additional dairy products, may not be necessary if the client's current protein intake is already sufficient, and it could contribute to excessive caloric and saturated fat intake. Option D, decreasing protein intake and eating more whole grains and vegetables, could be beneficial depending on the total protein and caloric intake of the client, but the statement about "half of the diet" from meat and eggs doesn't necessarily indicate an excessive protein intake without knowing the overall diet details. Thus, emphasizing an increase in fruits and vegetables without necessarily altering protein intake significantly is a balanced approach.

An adult with pneumonia is diaphoretic, tachycardia, and confused. The cardiac monitor indicates sinus tachycardia with frequent premature multifocal ventricular beats. Arterial blood gas (ABG) findings are: pH 7.26, PaCO2 66 mmHgl HCO3 23 mEq/L (22 mmol/L). Which intervention is most important for the nurse to include in this client's plan of care? A. Obtain a 12 lead electrocardiogram (ECG) daily B. Assess for apical-radial pulse deficit with vital signs C. Monitor the respiratory rate and depth continuously D. Maintain a patent IV catheter for antibiotic therapy

C. Monitor the respiratory rate and depth continuously The client's ABG findings indicate respiratory acidosis (pH 7.26, PaCO2 66 mmHg, HCO3 23 mEq/L), a condition where there is an accumulation of CO2 due to inadequate respiration. This can be due to the pneumonia affecting the client's lung function. The symptoms of diaphoresis, tachycardia, confusion, and the cardiac monitor showing sinus tachycardia with frequent premature multifocal ventricular beats also suggest that the client's respiratory status is compromised, affecting oxygenation and leading to these systemic symptoms. Continuous monitoring of the respiratory rate and depth is critical in this situation to assess the effectiveness of the respiratory interventions, detect further deterioration, and guide the need for additional interventions such as supplemental oxygen or mechanical ventilation. While obtaining a 12-lead ECG daily (A) can provide important information on the heart's electrical activity and is useful in monitoring for arrhythmias, it does not directly address the immediate problem of compromised respiratory function. Assessing for apical-radial pulse deficit with vital signs (B) is important for detecting cardiac issues but is not the priority given the ABG results. Maintaining a patent IV catheter for antibiotic therapy (D) is essential for treating the underlying pneumonia, but the immediate concern is ensuring adequate oxygenation and ventilation, which is directly addressed by monitoring respiratory rate and depth continuously.

When assessing a client who is returned to the unit following a left nephrectomy, then ruse notes a small amount of bloody drainage at the drain site. Which intervention is most important for the nurse to include in this client's plan of care? A. Provide sips of water and ice chips B. Turn, cough, deep breath C. Monitor urinary output hourly D. Document temperature every four hours

C. Monitor urinary output hourly After a nephrectomy (removal of a kidney), monitoring urinary output is crucial. It's important to ensure that the remaining kidney is functioning properly and capable of handling the body's filtration needs on its own. A small amount of bloody drainage at the drain site may be expected postoperatively, but changes in urinary output can indicate complications such as urinary tract obstruction, bleeding, or decreased function of the remaining kidney. Regular monitoring allows for early detection of these issues, enabling timely intervention if necessary.

A client receives a new prescription for a selective serotonin reuptake inhibitor (SSRI). Which information should the nurse include in the discharge teaching? A. Monitor blood pressure regularly B. Assess heart rate before each dose C. Report a decrease in sexual interest D. Avoid consumption of aged cheeses

C. Report a decrease in sexual interest SSRIs are commonly associated with sexual side effects, including decreased libido, anorgasmia, and erectile dysfunction. Clients should be informed about these potential side effects and encouraged to report them, as they can significantly impact quality of life and adherence to medication. Option A, monitoring blood pressure regularly, and Option B, assessing heart rate before each dose, are not typically necessary for clients taking SSRIs, as these medications do not commonly have significant effects on blood pressure or heart rate that require regular monitoring before each dose. Option D, avoiding consumption of aged cheeses, is relevant advice for clients taking monoamine oxidase inhibitors (MAOIs), not SSRIs. Aged cheeses contain tyramine, which can interact with MAOIs and lead to hypertensive crises, but this is not a concern with SSRIs.

The nurse orients a client with depression to a new room on the mental health unit. The client states, "It seems strange that I don't have a TV in my room." Which statement is best for the nurse to provide? A. It's important to be out of your room and talking to others. B. You can watch TV as much as you want outside of your room. C. Watching TV is a passive activity and we want you to be active. D. Sometimes clients feel like the TV is sending them messages.

C. Watching TV is a passive activity and we want you to be active. This response addresses the therapeutic goals of encouraging the client to engage in more active, participatory forms of therapy and interaction, which are more beneficial for mental health and recovery from depression. It gently educates the client on the rationale behind not having a TV in their room, emphasizing the importance of active participation in their recovery process. Option A is somewhat directive and doesn't explain the therapeutic rationale behind encouraging interaction with others. Option B may appear dismissive of the client's observation and doesn't provide a therapeutic rationale either. Option D might introduce an unnecessary concern for the client and is not universally applicable or relevant to the initial statement about the absence of a TV.

A client has a viral hepatitis A and hepatic encephalopathy. An unlicensed assistive personnel (UAP) is assigned to help the client with a bath. Which information should the nurse reinforce with the UAP? A. Don a mask when handling bedpan B. Maintain strict reverse isolation C. Wear gloves while giving the bath D. Restrict the client's family visitation

C. Wear gloves while giving the bath Hepatitis A is a liver infection caused by the hepatitis A virus (HAV), which is primarily transmitted through the fecal-oral route, either by direct contact with an infected person or by consuming contaminated food or water. Wearing gloves while providing personal care, such as giving a bath, helps prevent the spread of the virus and protects the UAP from potential exposure to bodily fluids that might contain HAV. Option A, donning a mask when handling the bedpan, is not specifically required for hepatitis A, as the primary route of transmission is not respiratory but fecal-oral. However, standard precautions should always be followed. Option B, maintaining strict reverse isolation, is not indicated for hepatitis A. Reverse isolation is used to protect immunocompromised clients from being infected by others, which is not the concern with hepatitis A. Option D, restricting the client's family visitation, is unnecessary for hepatitis A from an infection control perspective. Family members should be educated on proper hygiene practices, such as handwashing, to prevent the spread of the virus, but they do not need to be restricted from visiting.

Which statement by a client with systemic lupus erythematous (SLE) indicates the best understanding of proper skin care? A. "Tanning booths are okay to use but I need to stay out of direct sunlight" B. "I should apply drying agents on my draining lesions every day" C. "I need to make sure no one else comes in contact with my skin rash" D. "If I'm out in the sun, I need to use a very strong sun block"

D. "If I'm out in the sun, I need to use a very strong sun block" Clients with SLE are advised to protect their skin, as UV light can exacerbate the disease, including triggering skin lesions and other symptoms. using strong sunblock, with a high SPF, and reapplying it regularly can help minimize these effects and is a key part of managing SLE effectively.

A client with partial-thickness and full-thickness burns over 50% of the body is admitted to the Emergency Department. The healthcare provider prescribes hydromorphone 4 mg IM every 4 hours for pain. The client is complaining of "9" on the 1 to 10 pain scale. Which intervention should the nurse implement first? A. Contact the healthcare provider and question the prescription B. Assist the client with relaxation techniques and guided imagery C. Rule out any complications prior to administering the medication D. Administer an additional dose of the medication into an unburned area

D. Administer an additional dose of the medication into an unburned area In cases of severe pain, especially with burns, effective pain management is crucial. Intramuscular (IM) injections should be administered in an area of the body that is not affected by burns to ensure proper absorption of the medication and to avoid additional pain and damage to the burned tissues. The prescription for hydromorphone, an opioid analgesic, indicates that the healthcare provider has already assessed the need for potent pain management. Given the client's high pain score, administering the prescribed medication promptly is essential to alleviate suffering and improve comfort. While contacting the healthcare provider (A) might be necessary if the medication does not adequately control the pain, or if there are concerns about the prescription, it is not the first action to take when the client is in severe pain. Assisting the client with relaxation techniques and guided imagery (B) can be helpful as an adjunct to pharmacological pain management but is unlikely to be sufficient for severe pain relief on its own. Ruling out complications (C) is an ongoing process, but when a medication is specifically prescribed for pain management, the priority is to address the pain as efficiently as possible, assuming no immediate contraindications to the medication's use are present.

A client has been on bed rest following a cerebrovascular accident or stroke that occurred two days ago. On the third day, the plan of care includes getting the client out of bed and into a bedside chair. Prior to assisting the client out of bed for the first time, which action should the nurse take? A. Offer the client to void before getting out of bed B. Assess the client''s brain stem reflexes C. Perform pupillary response assessment D. Assess the client's blood pressure

D. Assess the client's blood pressure Assessing the client's blood pressure is crucial to identify any potential issues with orthostatic hypotension, which is a common and significant risk when a client first gets out of bed after being in a supine position for an extended period. Orthostatic hypotension can lead to dizziness, fainting, and falls, posing a significant risk of injury, especially in clients recovering from a stroke who may already have mobility and balance impairments. Knowing the client's blood pressure can help the nurse plan the move safely, ensuring that interventions to mitigate the risk of orthostatic hypotension are in place if needed. While offering the client to void before getting out of bed (A) is a practical consideration for comfort and to prevent accidents, it does not have the same immediate safety implications as assessing blood pressure. Assessing the client's brain stem reflexes (B) and performing a pupillary response assessment (C) are important neurological assessments post-stroke, but they do not directly address the immediate risks associated with transferring the client to a chair.

The nurse is teaching a client about coronary artery disease (CAD) preventative health. Which behavior stated by the client indicates a need for additional information and teaching? A. Increased fiber, complex carbohydrates, and vegetables in the diet B. Performs 30 minutes of physical activity 5 days per week C. Avoids foods that are high in saturated fats D. Decreased the number of cigarettes smoked per day

D. Decreased the number of cigarettes smoked per day For coronary artery disease (CAD) prevention, the goal for smoking is complete cessation, not just a reduction in the number of cigarettes smoked per day. Smoking is a major risk factor for CAD, and even smoking a few cigarettes a day can significantly increase the risk of heart disease. The client's statement implies a misunderstanding of the importance of completely stopping smoking to reduce the risk of CAD. Additional teaching should focus on the benefits of quitting smoking entirely and providing resources or strategies to help the client quit smoking.

A young adult female visits the clinic for primary dysmenorrhea and tells the nurse that she starts taking a calcium supplement to reduce her menstrual cramps, but quit taking the calcium because it caused constipation. The client wants to know what can she do to relieve her menstrual cramps. Which action should the nurse implement first to address the client's concern? A. Ask her how much calcium she had been taking daily B. Question the client about her use of birth control pills C. Encourage client to increase her dietary intake of fiber D. Determine if she takes any over-the-counter analgesics

D. Determine if she takes any over-the-counter analgesics This initial step helps to assess the client's current pain management strategies and provides a basis for recommending appropriate and immediate interventions for pain relief. Over-the-counter (OTC) analgesics, such as nonsteroidal anti-inflammatory drugs (NSAIDs), are commonly used and effective for managing the pain associated with primary dysmenorrhea. Understanding whether the client already uses these medications, and how she uses them, can guide further advice on effective pain management techniques, potential adjustments in dosing, or the timing of medication for optimal relief. The other options, while potentially useful in the broader context of managing symptoms of dysmenorrhea or addressing side effects of supplements, do not directly address the immediate concern of pain relief as effectively as understanding the client's current use of pain medication.

A male client with a fungal infection of the toenail reports to the nurse that he has been applying an over-the-counter triple antibiotic ointment to the infection daily for two weeks without any improvement. Which action should the nurse take? A. Suggest that the client use the ointment twice a day to be more effective B. Advise the client to obtain a prescription-strength formulation of the ointment C. Reassure the client that treatment of fungus-infected toenails often takes several months D. Instruct the client to obtain a prescription for oral terbinafine

D. Instruct the client to obtain a prescription for oral terbinafine Fungal infections of the toenail (onychomycosis) are not effectively treated with antibiotics since antibiotics target bacteria, not fungi. Terbinafine is an antifungal medication that can be taken orally and is specifically used for treating fungal infections of the nails. Recommending that the client obtain a prescription for oral terbinafine is appropriate because it targets the fungal infection directly, whereas the over-the-counter triple antibiotic ointment would not be effective against fungal pathogens.

A nurse arrives at the scene of a motor vehicle collision, and finds that the male driver is sitting in the car screaming about severe knee pain. He is alert and oriented, but his knee is crushed into the dashboard. A bystander reports that a 911 call has been made and help is on the way. What intervention should the nurse implement? A. Place a rolled pillow behind the driver's neck to stabilize the spinal cord. B. Carefully assist the driver out of the car to assess and treat the injury. C. Tell the driver to try to see if he can move his toes on the affected leg. D. Instruct the driver to stay in the car and try not to move unnecessarily.

D. Instruct the driver to stay in the car and try not to move unnecessarily In a scenario involving trauma, especially with a potential crush injury to the knee and the individual still inside a vehicle, it's crucial to prevent further injury until emergency medical services (EMS) arrive. Moving the individual or attempting to assess and treat injuries without proper immobilization equipment and support could exacerbate the injury, especially if there are underlying fractures or spinal injuries that are not immediately apparent. The nurse should ensure the driver remains as still as possible to avoid further harm. Option A, placing a rolled pillow behind the neck, is not advised without assessing for spinal injuries first, and in this scenario, the priority is to keep the driver still to avoid exacerbating the knee injury or any potential unseen injuries. Option B, assisting the driver out of the car, could be very dangerous without a proper assessment by emergency services and the right equipment to stabilize the injury. Option C, telling the driver to move his toes, could be part of an assessment for neurological function, but in this context, instructing the driver to stay still and avoid unnecessary movements is prioritized to prevent worsening of injuries.

The nurse is providing wound care to a client with a stage 3 pressure ulcer that has large amount of eschar. The wound care prescription states, "Clean the wound and then apply collagenase." Collagenase is a deriding agent. The prescription does not specify a cleansing method. Which technique should the nurse use to cleanse the pressure ulcer? A. Lightly coat the wound with providone-iodine solution B. Remove the eschar with a wet-to-dry dressing C. Flush the wound with sterile hydrogen peroxide D. Irrigate the wound with sterile normal saline

D. Irrigate the wound with sterile normal saline This method is gentle, effective in cleaning the wound bed, and does not introduce substances that could inhibit the action of the collagenase or harm the healing tissue. Normal saline is isotonic and does not damage or dry out new tissue, making it the preferred choice for wound irrigation in most clinical settings, including the care of pressure ulcers. Using providone-iodine solution, hydrogen peroxide, or wet-to-dry dressings could be harmful or too harsh for the wound. Providone-iodine and hydrogen peroxide can damage new granulation tissue and impede wound healing. Wet-to-dry dressings, while used for mechanical debridement, can also be painful and potentially damaging to new tissue. Therefore, the best option is to irrigate the wound with sterile normal saline.

A newborn with myelomeningocele is admitted to the neonatal intensive care unit. Which preoperative nursing intervention should the nurse implement first? A. Measure the head circumference while in prone position B. Apply an antibiotic ointment to the exposed area C. Apply a diaper below the myelomeningocele D. Place the infant on the abdomen to protect the sac

D. Place the infant on the abdomen to protect the sac Positioning the infant prone (on the abdomen) is a priority to prevent pressure on the myelomeningocele sac, which can lead to rupture and increase the risk of infection. This position helps protect the sac from trauma and reduces the risk of further damage to the exposed spinal cord and nerves. Measuring head circumference (A) is important for monitoring hydrocephalus, a common complication of myelomeningocele, but it does not take precedence over protecting the integrity of the myelomeningocele sac. Applying antibiotic ointment (B) might be part of the care plan to prevent infection, but direct application to the sac without specific medical orders could be harmful, and initial care focuses on preventing trauma and infection through protective positioning and sterile covering. Applying a diaper below the myelomeningocele (C) is also important to prevent contamination with urine or feces, but it follows the immediate priority of positioning the infant to safeguard the sac.

The nurse is preparing a client for discharge who was recently diagnosed with Addison's disease. Which instruction is most important for the nurse to include in this client's discharge teaching plan? A. Use a walker when weakness occurs B. Avoid extreme environmental temperatures C. Increase daily intake of sodium in diet D. Take prescribed cortisone accurately

D. Take prescribed cortisone accurately Addison's disease is characterized by adrenal insufficiency, where the adrenal glands do not produce enough of certain hormones, including cortisol. Cortisol is essential for regulating various body functions, including metabolism and the body's response to stress. Taking prescribed corticosteroid medication, such as cortisone, is crucial for managing Addison's disease, as it replaces the cortisol the body is not able to produce. Accurate adherence to medication regimen is essential to prevent an adrenal crisis, a life-threatening condition that can occur if the body is under stress and does not have enough cortisol. While the other instructions are also important for the overall management and well-being of the client: B. Avoiding extreme environmental temperatures and C. Increasing daily intake of sodium in the diet are part of managing Addison's disease, as individuals with this condition can be more sensitive to temperature extremes and may require more sodium due to aldosterone deficiency. A. Using a walker when weakness occurs addresses a symptom of the disease but does not directly manage the underlying condition like taking prescribed cortisone does. Ensuring that the client understands the importance of adhering to their corticosteroid medication regimen is critical for preventing complications and ensuring long-term stability of the condition.

A gravida 2 para 3 who is Rh-negative delivers a full-term infant at home with the assistance of a nurse midwife. Two days later, the client calls the clinic to ask if it is necessary to see the healthcare provider since the infant is healthy, and she is not having any complications. The woman's history indicates that both previously born infants were Rh-negative. Which response should the nurse provide? A. Rho (D) immune globulin injections must be administered within 24 hours after delivery B. It is likely that the husband is Rh-negative, and if so Rho (D) immune globulin injection is not needed C. Rho (D) immune globulin is not indicated since both previous babies were Rh-negative D. The newborn's blood type should be tested to determine the need for Rho (D) immune globulin injection

D. The newborn's blood type should be tested to determine the need for Rho (D) immune globulin injection For an Rh-negative mother, the administration of Rho(D) immune globulin after delivery is essential if the baby is Rh-positive. This helps prevent the mother from developing antibodies that could affect future pregnancies. Even though the mother's previous infants were Rh-negative, each pregnancy and baby can differ in terms of Rh status. Testing the newborn's blood type is crucial to determine the need for Rho(D) immune globulin injection, ensuring proper postpartum care for the mother.

A home-bond client with severe, end-stage chronic obstructive pulmonary disease (COPD) is being visited by the home health. nurse. Which instruction should the nurse include in the client's teaching plan? A. Use the beta-agonist inhalers q2h, around the clock B. Use pursed-lip breathing techniques continually, around the clock C. Cluster activities together, first thing in the morning D. Use oxygen continuously, at the lowest dose possible

D. Use oxygen continuously, at the lowest dose possible Continuous oxygen therapy can help maintain adequate oxygen levels, reduce the workload of the heart, and improve the quality of life for clients with end stage COPD. The goal is to keep oxygen saturation above a certain level (usually at least 88-92%) to minimize symptoms of hypoxemia and reduce complications associated with low oxygen levels. Using the lowest effective dose reduces the risk of oxygen toxicity and CO2 retention, which can be a concern in COPD patients.

A patient has a herniated intervertebral lumbar disc and is experiencing right leg pain. How should the patient get up from the bed?

Lie on side to get out of bed/ Turn to the side and use hands to prop themselves up from the bed

A client with newly diagnosed Crohn's disease asks the nurse about dietary restrictions. How should the nurse respond? A. Describe the use of an elimination diet to find trigger foods B. Explain that the need to restrict fluids is the primary limitation C. Instruct the client to avoid foods with gluten, such as wheat bread D. Advise the client to limit foods that are high in calcium and iron

a. describe the use of an elimination diet to find trigger foods Crohn's disease is an inflammatory bowel disease that can affect any part of the gastrointestinal tract, and symptoms can vary significantly among individuals. Dietary management is personalized and involves identifying and avoiding foods that exacerbate symptoms. An elimination diet helps in this process by removing certain foods believed to trigger symptoms and then gradually reintroducing them to determine which foods cause issues for the individual. This approach allows the person with Crohn's disease to tailor their diet to minimize symptoms while still ensuring nutritional needs are met. Option B, explaining that the need to restrict fluids is the primary limitation, is not accurate. Fluid intake is generally encouraged in Crohn's disease to prevent dehydration, especially during flare-ups. Option C, instructing the client to avoid foods with gluten, such as wheat bread, might be necessary for clients with a concurrent diagnosis of celiac disease, but gluten is not universally a trigger for Crohn's disease symptoms. Option D, advising the client to limit foods that are high in calcium and iron, is not typically recommended unless there's a specific reason related to the individual's health status. In fact, individuals with Crohn's disease often need more of these nutrients due to malabsorption issues associated with their condition.

A toddler is hospitalized with Kawasaki's disease. Pharmacological management includes aspirin therapy. Which is the primary benefit of the aspirin? A. Minimize vascular inflammation B. Control high fever C. Reduce joint swelling D. Manage irritability

a. minimize vascular inflammation Kawasaki's disease is a condition that primarily affects children and involves inflammation of the blood vessels throughout the body, including the coronary arteries. Aspirin, as an anti-inflammatory medication, helps reduce the inflammation of the blood vessels, thereby minimizing the risk of developing coronary artery aneurysms, which is a serious complication of Kawasaki's disease. While aspirin may also help control fever and reduce some symptoms like joint swelling, its main therapeutic role in Kawasaki's disease is the prevention of coronary artery complications through its anti-inflammatory effects.

An elderly client with Alzheimer's disease is being discharged with several prescriptions and the nurse plants to teach the daughter, who is the primary caregiver, about drug administration. What is the best method to use in evaluating the daughter's understanding? A. Observe the daughter administering the medication before leaving the hospital B. Evaluate the questions asked by the daughter after providing them with written instructions C. Administer a written test on medication administration after watching the nurse administer the drugs. D. Ask the daughter to repeat the dosages and methods for administration to assess her understanding

a. observe the daughter administering the medication before leaving the hospital This hands-on approach allows the nurse to directly assess the daughter's ability to administer the medications correctly and safely. It also provides an opportunity for immediate feedback and correction of any errors, ensuring the caregiver is competent in managing the patient's medication regimen. This method is more effective and practical than written tests or solely verbal repetition, as it assesses the ability to perform the task in a real-world setting.

The nurse is assessing a client who has a bowel obstruction. Which observations should the nurse expect to find? (Select all that apply) A. Peristaltic waves observed B. Abdominal distention C. Abdomen soft on palpation D. Dullness on percussion E. High pitched bowel sounds

a. peristaltic waves observed b. abdominal distention e. high pitched bowel sounds visible peristaltic waves may be seen, especially in thin individuals, as the intestine attempts to move the contents past the obstruction this is a common finding due to the accumulation of intestinal contents and gas above the level of the obstruction early in the obstruction, bowel sounds may be high pitched and more frequent than normal due to the intestines attempting to move the contents past the obstruction

A 6-year-old male with a body mass index (BMI) in the 95th percentile for gender and age arrives at the clinic after a referral from the school nurse. His laboratory findings include hemoglobin A1c of 5.5% (0.06), blood pressure (BP) in the 50th percentile for age, height in the 75th percentile, and an LDL cholesterol of 90 mg/dL (2.33 mmol/L). Which lifestyle modification should the nurse discuss with the parents? A. Recommend increasing daily fruits and vegetables and daily exercise B. See a healthcare provider to further assess for diabetes and hypertension C. Return in one month for another evaluation of serum lipids and blood pressure D. Instruct the parents to weight the child weekly and measure his BP daily

a. recommend increasing daily fruits and vegetables and daily exercise Given the child's BMI in the 95th percentile, indicating overweight or obesity, focusing on healthy eating and increasing physical activity are key strategies to help manage his weight and reduce the risk of developing associated health conditions. The hemoglobin A1c of 5.5% is within a normal range, and an LDL cholesterol of 90 mg/dL is also within acceptable limits for children. The blood pressure in the 50th percentile is normal for his age. These factors suggest that lifestyle modifications aimed at achieving a healthier weight through diet and exercise are the most appropriate initial steps. Regular follow-up to monitor his progress and any changes in his health status would be advisable, but the immediate focus should be on dietary improvements and physical activity.

An adolescent client who has been lacto-ovo-vegetarian for six months tells the nurse about experiencing increasing fatigue. What recommendation should the nurse provide? A. Eliminate carbonated soft drinks from the diet B. Eat spinach three times a week C. Increase intake of fruit to 6 servings per day D. Decrease saturated fat intake

b. eat spinach three times a week For an adolescent client experiencing fatigue and following a lacto-ovo-vegetarian diet, increasing intake of iron-rich foods like spinach can be beneficial. Spinach is a good source of non-heme iron, which is especially important in vegetarian diets to prevent iron-deficiency anemia, a common cause of fatigue. Additionally, foods rich in vitamin C can enhance the absorption of non-heme iron, so it might be helpful to consume vitamin C-rich foods (like citrus fruits, tomatoes, and bell peppers) alongside iron-rich meals.

A client who is currently receiving methadone 10 mg q8 hours reports that pain is an 8 on a scale of 1 to 10. Which action should the nurse implement? A. Explore use of non-pharmacological interventions B. Give a PRN dose of an analgesic for breakthrough pain C. Administer the next scheduled dose of methadone early D. Tell client when the next dose of methadone is scheduled

b. give a PRN dose of an analgesic for breakthrough pain Methadone is a long-acting opioid used for chronic pain management and opioid replacement therapy. When a client reports high levels of pain despite being on a scheduled opioid regimen, administering a prescribed as-needed (PRN) dose of an analgesic for breakthrough pain is appropriate. This approach provides immediate pain relief until the next scheduled dose of the long-acting medication (methadone, in this case) is due. It's important to have orders for breakthrough pain medication as part of a comprehensive pain management plan. Option A, exploring the use of non-pharmacological interventions, is valuable as part of a holistic approach to pain management but may not be sufficient for immediate relief of severe pain rated as an 8. Option C, administering the next scheduled dose of methadone early, is generally not advisable without consulting with a healthcare provider, as it may lead to dosing errors or increased risk of opioid toxicity. Option D, telling the client when the next dose of methadone is scheduled, does not address the immediate need for pain relief and can be perceived as dismissive of the client's current pain experience.

A middle-aged client newly diagnosed with cholelithiasis is choosing the evening meal. What food choice should the nurse encourage the client to omit? A. Ketchup B. Ice cream C. Bread D. Beef broth

b. ice cream Cholelithiasis, or gallstones, can lead to gallbladder attacks when high-fat foods are consumed, as fat triggers the gallbladder to contract, which can cause pain if gallstones block the bile ducts. Ice cream is high in fat and could potentially trigger a gallbladder attack in individuals with cholelithiasis. Encouraging a diet low in fat can help manage symptoms and reduce the risk of painful episodes. The other food choices listed: A. Ketchup, C. Bread, and D. Beef broth, are less likely to cause issues compared to high-fat foods like ice cream. However, individual tolerances can vary, and overall dietary patterns should focus on low-fat, balanced meals to manage cholelithiasis effectively.

When entering the room of a sedated postoperative client, which assessment requires the most immediate intervention by the nurse? A. The urinary catheter drainage bag is almost completely full of amber urine B. Low intermittent suction prescribed for the nasogastric tube is turned off C. A Hemovac drain is partially full of serous drainage and is not compressed D. Oxygen is being administered via nasal cannula at 4 L/min without humidification

b. low intermittent suction prescribed for the nasogastric tube is turned off A nasogastric tube with prescribed low intermittent suction is typically used to remove gastric contents and prevent nausea, vomiting, distension, and potential aspiration, especially in postoperative clients. If the suction is turned off, it can lead to accumulation of gastric contents, increasing the risk for aspiration, especially in a sedated client who may have impaired protective airway reflexes. Restoring the prescribed suction is critical to prevent these complications. While the other assessments also need attention, they are not as immediately critical as ensuring the nasogastric suction is functioning as prescribed: A. A urinary catheter drainage bag that is almost full needs to be emptied to prevent backflow and infection, but this is not as immediately life-threatening as potential aspiration. C. A Hemovac drain that is partially full of serous drainage and is not compressed needs to be emptied and recompressed to continue effective drainage, but this does not pose an immediate threat to the client's airway or risk of aspiration. D. Oxygen being administered without humidification may lead to dry mucous membranes over time, but correcting this does not carry the same urgency as preventing aspiration and maintaining a clear airway.

The nurse assesses a client who is newly diagnosed with hyperthyroidism and observes that the client's eyeballs are protuberant, causing a wide-eyed appearance and eye discomfort. Based on this finding, which action should the nurse include in this client's plan of care? A. Assess for signs of increased intracranial pressure B. Obtain a prescription for artificial tear drops C. Prepare to administer intravenous levothyroxine D. Review the client's serum electrolyte values

b. obtain a prescription for artificial tear drops The described symptoms are indicative of Graves' ophthalmopathy (also known as thyroid eye disease), a condition associated with hyperthyroidism, particularly Graves' disease. This condition can cause symptoms such as dryness, grittiness, and irritation due to the protrusion of the eyeballs and exposure of more of the eye surface to the air. Artificial tear drops can help alleviate these symptoms by moisturizing the eyes and providing relief from dryness and discomfort. Option A, assessing for signs of increased intracranial pressure, may not be directly related to the symptoms described, as the eye changes are more characteristic of Graves' ophthalmopathy rather than increased intracranial pressure. Option C, preparing to administer intravenous levothyroxine, would not be indicated in this situation, as levothyroxine is used to treat hypothyroidism, not hyperthyroidism. In fact, treatment for hyperthyroidism would aim to reduce thyroid hormone levels, not increase them. Option D, reviewing the client's serum electrolyte values, while important in overall patient care, does not directly address the immediate concern of eye discomfort associated with the thyroid condition.

The healthcare provider prescribes 5% Dextrose Injection, USP with 20 units of regular insulin for a client with a serum potassium level of 6.0 mEq/L (6.0 mmol/L SI) and glucose level of 180 mg/dL (10.0 mmol/L SI). Which evaluation is most important for the nurse to include in this client's plan of care? A. Monitor and document strict intake and output B. Evaluate glucose levels before and after meals C. Assess the serum potassium level q4 hours D. Obtain a 12-lead electrocardiogram daily

c. assess the serum potassium level q4 hours This intervention is critical because the primary goal of administering insulin in this context is to lower the serum potassium level by facilitating the shift of potassium from the extracellular fluid into the cells. High serum potassium levels (hyperkalemia) can lead to serious cardiac arrhythmias and other complications. Regular monitoring of the serum potassium level is essential to ensure that it returns to and remains within the normal range and to prevent hypokalemia (too low potassium level) as a result of the treatment. Glucose levels, fluid balance, and cardiac monitoring are also important but secondary to the immediate need to correct the potassium imbalance in this scenario.

A young adult visits the clinic reporting symptoms associated with gastritis. Which information in the client's history is most important for the nurse to address in the teaching plan? A. Recently became a vegetarian and eats a lot of high fiber foods B. Snacks on foods with very high salt content on a daily basis C. Consumes 10 or more drinks of alcohol every weekend D. Exercises vigorously every evening right before going to bed

c. consumes 10 or more drinks of alcohol every weekend Gastritis, an inflammation of the stomach lining, can be caused by various factors, including excessive alcohol consumption. Alcohol can irritate and erode the stomach lining, leading to inflammation, pain, and other symptoms of gastritis. Addressing the client's high intake of alcohol is crucial in managing and preventing further irritation to the stomach lining. The nurse's teaching plan should focus on reducing alcohol consumption as a key component of managing gastritis symptoms and promoting overall gastrointestinal health.

A newborn with a respiratory rate of 40 breaths/minute at one minute after birth is demonstrating cyanosis of the hands and feet. What action should the nurse take? A. Assess bowel sounds B. Rub the infant's back C. Continue to monitor D. Assist with intubation

c. continue to monitor A respiratory rate of 40 breaths per minute is normal for a newborn, and cyanosis of the hands and feet (acrocyanosis) is a common finding in the first few hours after birth due to immature circulation. This condition typically resolves on its own as the newborn's circulation adjusts to life outside the womb. Continuous monitoring is appropriate to ensure that the newborn's respiratory status remains stable and that the cyanosis resolves as expected.

The nurse notes that a client's plan of care includes the problem, "Deficient Knowledge (Dietary changes)." In developing a teaching plan, what information is most important for the nurse to obtain? A. Etiology of the problem B. Availability of the dietician for consultation C. Family members involved in the client's care D. Age of the client

c. family members involved in the client's care Including family members who are involved in the client's care is crucial for several reasons. First, they often play a significant role in meal preparation and food choices, especially if the client is not fully independent. Second, family support can enhance the client's motivation and adherence to dietary changes. Third, educating family members ensures that they understand the rationale behind the dietary changes, which can help them make supportive decisions regarding food purchases and meal preparation. This approach fosters a supportive environment for the client to make and sustain the necessary dietary adjustments. While the etiology of the problem (A) and the age of the client (D) are important for understanding the context and tailoring the education, they do not directly influence the immediate planning and implementation of dietary teaching as much as knowing who will be directly involved in supporting the client's dietary changes. The availability of the dietician for consultation (B) is also valuable for providing expert advice, but the involvement of family members is a more critical factor for the practical application and ongoing support of dietary changes in the client's daily life.

The nurse is caring for a newborn who is 18 inches long, weighs 4 pounds, 14 ounces (2.2 kg), has a head circumference of 13 inches (33 cm), and a chest circumference of 10 inches (25.4 cm). Based on these physical findings, assessment for which condition has the highest priority? A. Polycythemia B. Hyperbilirubinemia C. Hypoglycemia D. Hyperthermia

c. hypoglycemia The newborn's weight of 4 pounds, 14 ounces (2.2 kg) indicates low birth weight, which is a common risk factor for hypoglycemia in newborns. Low birth weight babies have less glycogen stored in their livers, making them more prone to developing low blood sugar levels, especially in the first few hours to days of life. Early identification and management of hypoglycemia are crucial to prevent possible complications such as neurological damage. While polycythemia (A) and hyperbilirubinemia (B) can also be concerns in newborns, particularly those with low birth weight or other risk factors, the immediate risk of hypoglycemia and its potential consequences typically take priority in the initial assessment and management. Hyperthermia (D) is less likely to be a priority given the provided measurements and the common concerns associated with low birth weight infants.

A client receives a new prescription for a selective serotonin reuptake inhibitor (SSRI). Which information should the nurse include in the discharge teaching? A. Monitor blood pressure regularly B. Assess heart rate before each dose C. Report a decrease in sexual interest D. Avoid consumption of aged cheeses

c. report a decrease in sexual interest When a client receives a new prescription for a selective serotonin reuptake inhibitor (SSRI), it's important to inform them about potential side effects, including sexual side effects such as decreased libido, difficulty achieving orgasm, or erectile dysfunction. These side effects can affect quality of life and potentially impact adherence to medication. Clients should be encouraged to report these or any other side effects to their healthcare provider, as adjustments to the treatment plan may be necessary. Options A and B are not typically associated with SSRIs. While SSRIs can have some effect on blood pressure and heart rate, regular monitoring of these before each dose is not standard practice for clients on SSRIs. Option D, avoiding consumption of aged cheeses, is related to dietary restrictions for clients taking monoamine oxidase inhibitors (MAOIs), not SSRIs. MAOIs can interact with tyramine-rich foods like aged cheeses, leading to hypertensive crises, but this is not a concern with SSRIs.

A nurse is speaking with a client who is addicted to heroin and who just learned that she is pregnant. The client states, "I just started taking methadone. Is there anything else I can do to make sure my baby is healthy?" Which information should the nurse provide? A. Describe genetic testing protocols B. Sign up for group therapy sessions C. Start a prenatal care plan as soon as possible D. Discontinue the methadone right away

c. start a prenatal care plan as soon as possible For a pregnant client addicted to heroin and starting methadone treatment, the most beneficial action is to start a prenatal care plan as soon as possible. Prenatal care is crucial for monitoring the health of both the mother and the baby throughout the pregnancy. It allows healthcare providers to offer specific advice on managing methadone treatment safely during pregnancy and to monitor for any potential complications. Methadone maintenance therapy is a standard treatment for opioid dependence in pregnant women, as it stabilizes the maternal environment, reducing the risk of obstetric complications and improving outcomes for the neonate. Discontinuing methadone abruptly can be harmful to both the mother and the fetus, leading to withdrawal symptoms that may endanger the pregnancy.

Which information should the nurse include when giving discharge instructions to a client following a left eye cataract extraction with lens implant? A. Turn, cough, and deep breathe every 2 hours B. Observe pupil response of the right eye C. Sleep flat in a supine position D. Administer a stool softener

d. administer a stool softener This recommendation is important because it helps to avoid straining during bowel movements, which can increase intraocular pressure and potentially harm the surgical site or affect the healing process. Keeping the stool soft and ensuring easy bowel movements can help prevent complications related to increased pressure in the eye. Option A, "Turn, cough, and deep breathe every 2 hours," is generally advised to prevent respiratory complications post-surgery, but it's not specific or particularly crucial following cataract surgery. Option B, "Observe pupil response of the right eye," while important in assessing neurological status, doesn't directly relate to the care of the left eye after cataract surgery. Option C, "Sleep flat in a supine position," may not be advisable depending on the surgeon's specific instructions, as some clients may be instructed to avoid lying on the side of the surgery to prevent pressure on the eye; position recommendations can vary.

A client reports epigastric pain after receiving an oral dose of ketolorac. Which action should the nurse implement? A. Offer the client a PRN dose of ibuprofen B. Ask if the client is taking home remedies C. Administer subsequent doses with meals D. Continue to monitor the client for GI distress

d. continue to monitor the client for GI distress Ketorolac can cause GI irritation, leading to symptoms like epigastric pain. The priority is to monitor the client for further GI distress and report the symptoms to the healthcare provider for potential adjustment of the medication regimen or addition of a gastroprotective agent if necessary. Option A, offering the client a PRN dose of ibuprofen, is not appropriate because ibuprofen is another NSAID that can further irritate the GI tract and exacerbate the client's symptoms. Option B, asking if the client is taking home remedies, while it might provide useful information about other potential sources of GI irritation, does not directly address the immediate concern of NSAID-induced epigastric pain. Option C, administering subsequent doses with meals, can help minimize GI irritation from NSAIDs in some cases, but it may not be sufficient if the client is already experiencing significant distress. Additionally, this action should be guided by the healthcare provider's instructions and considering the client's current symptoms and overall condition.

A client reports epigastric pain an hour after receiving an oral dose of ketolorac. Which action should the nurse implement? A. Offer the client a PRN dose of ibuprofen B. Ask if the client is taking home remedies C. Administer subsequent doses with meals D. Continue to monitor the client for GI distress

d. continue to monitor the client for GI distress Ketorolac can cause GI irritation, leading to symptoms like epigastric pain. The priority is to monitor the client for further GI distress and report the symptoms to the healthcare provider for potential adjustment of the medication regimen or addition of a gastroprotective agent if necessary. Option A, offering the client a PRN dose of ibuprofen, is not appropriate because ibuprofen is another NSAID that can further irritate the GI tract and exacerbate the client's symptoms. Option B, asking if the client is taking home remedies, while it might provide useful information about other potential sources of GI irritation, does not directly address the immediate concern of NSAID-induced epigastric pain. Option C, administering subsequent doses with meals, can help minimize GI irritation from NSAIDs in some cases, but it may not be sufficient if the client is already experiencing significant distress. Additionally, this action should be guided by the healthcare provider's instructions and considering the client's current symptoms and overall condition.

To control asthma, a client in a residential treatment facility uses a fluticasone propionate and salmetrol discus inhalation system, which provides inhaled powdered form of these combined medications. Which instruction should the nurse provide to this client's caregivers? A. Clients using the discus may experience decreased blood pressure B. Offer the discus to the client for use during an acute asthma attack C. When using the discus, have the client breathe out rapidly into the mouthpiece D. Explain that the client should not use the discus more than twice daily

d. explain that the client should not use the discus more than twice daily The fluticasone propionate and salmeterol discus inhalation system is a combination medication used for long-term control of asthma, not for relief of acute symptoms. It typically should be used not more than twice daily, according to prescribed intervals. This instruction ensures the medication is used correctly to manage asthma effectively while minimizing the risk of side effects from overuse.

An older client with type 2 diabetes mellitus presents to the Emergency Department with a respiratory infection. The nurse recognizes that the client is at risk for hyperosmolar hyperglycemic state (HHS) as a result of what process? A. Stress-induced release of hormones B. Severe greater than 103 F (39.4 C) C. Elevated white blood cell count D. Adverse reaction to IV antibiotics

A. Stress-induced release of hormones During periods of stress, such as with an infection, the body releases stress hormones (e.g., cortisol, epinephrine, and glucagon) that increase blood glucose levels by promoting gluconeogenesis and glycogenolysis and reducing glucose uptake by the cells. In individuals with diabetes, especially type 2 diabetes, the ability to compensate for this hyperglycemia is impaired, which can lead to significantly elevated blood glucose levels. In the context of an infection like a respiratory infection, the increased blood glucose can lead to dehydration and a further increase in blood glucose concentrations, culminating in HHS, a serious condition characterized by extreme hyperglycemia, dehydration, and altered consciousness. The other options, while they can be associated with complications in a diabetic patient, are not directly responsible for the pathophysiological process leading to HHS: B. Fever (Severe greater than 103 F [39.4 C]) can contribute to the body's stress response but is a symptom rather than a cause. C. An Elevated white blood cell count indicates an infection or stress response but is not a direct cause of HHS. D. An Adverse reaction to IV antibiotics could potentially complicate the clinical picture but is not a primary factor in the development of HHS.

The nurse is teaching a client with cancer about skin care for the portal site receiving external beam radiation. Which client action about skin care indicates a need for further teaching? A. Washes the radiation site with antibacterial soap and water B. Dries the area with patting motions after taking a shower C. Applies prescribed lotions to the radiation site D. Wears clothing to cover the radiation site

A. Washes the radiation site with antibacterial soap and water Clients receiving external beam radiation are generally advised to gently clean the treatment area with mild soap and water. Using antibacterial soap, which can be harsh and drying, might irritate the skin further. The emphasis is on gentle skin care to avoid irritation to the radiation site. It is important to use only mild, non-irritating soaps and to avoid any products that can dry out or irritate the skin, unless specifically recommended by the healthcare provider.

A client who recently returned from the Philippines presents to the clinic with high fever, chills, headache, and muscular aches. The healthcare provider suspects severe acute respiratory syndrome (SARS). To prevent transmission of the virus to others, which instruction is most important for the nurse to provide? A. Wear a N95 respirator mask during direct contact with others B. Use meticulous hand washing technique C. Discard all tissues used during coughing in a biohazard bag D. Cover mouth and turn head when coughing

A. Wear a N95 respirator mask during direct contact with others SARS is known to spread through close person-to-person contact, and respiratory droplets produced when an infected person coughs or sneezes can be inhaled by people nearby. Therefore, wearing an N95 respirator mask, which is designed to achieve a very close facial fit and very efficient filtration of airborne particles, is crucial for preventing the inhalation of the virus. This is especially important in healthcare settings where direct contact with the patient is necessary. While meticulous hand washing (B), discarding tissues used during coughing in a biohazard bag (C), and covering the mouth and turning the head when coughing (D) are all important infection control measures, the use of an N95 respirator mask directly addresses the primary mode of SARS transmission and offers the highest level of protection for individuals in close contact with the patient.

An overweight, young adult who was recently diagnosed with type 2 diabetes mellitus is admitted for a hernia repair. The client reports feeling very weak and jittery. Which actions should the nurse implement? (Select all that apply) A. Assess skin temperature and moisture B. Document anxiety on the surgical checklist C. Check finger stick glucose level D. Measure pulse and blood pressure E. Administer a PRN dose of regular insulin

A. assess skin temperature and moisture C. check finger stick glucose level D. measure pulse and blood pressure A: this assessment helps determine signs of hypoglycemia, which can cause sweating and cool skin. it's important in evaluating the client's immediate physiological status C: given the client's symptoms of weakness and jitteriness, along with their diagnosis of type 2 diabetes, checking the blood glucose level is crucial to determine if these symptoms are due to hypoglycemia, which requires immediate treatment. D: symptoms of weakness and jitteriness can also be related to cardiovascular responses. measuring vital signs helps in assessing the client's hemodynamic status and can indicate if the jitteriness is due to hypoglycemia or other causes like anxiety or pain

A patient has developed ABO incompatibility after several blood transfusions

ACUTE HEMOLYTIC REACTION


Kaugnay na mga set ng pag-aaral

Chapter 45- Antiarrhythmic Agents

View Set

nursing 6 unit 3 Brunner med surg Chapter 69: Management of Patients With Neurologic Infections, Autoimmune Disorders, and Neuropathies

View Set

14. ASU - CIS105 - Computer Applications and IT - Mid-term Exam

View Set